Difference between revisions of "2008 AIME I Problems/Problem 14"

(solution by azjps)
 
(Alternate Solution by chickendude)
Line 27: Line 27:
 
<cmath>BP^2 \le 405 + 729 \cdot \frac{1}{27} = \boxed{432}</cmath>
 
<cmath>BP^2 \le 405 + 729 \cdot \frac{1}{27} = \boxed{432}</cmath>
 
Equality holds when <math>x = 27</math>.
 
Equality holds when <math>x = 27</math>.
 +
 +
==Alternate Solution==
 +
<asy>
 +
unitsize(4mm);
 +
pair B=(0,13.5), C=(23.383,0);
 +
pair O=(7.794, 9), P=(2*7.794,0);
 +
pair T=(7.794,0), Q=(0,0);
 +
pair A=(2*7.794,4.5);
 +
 +
draw(Q--B--C--Q);
 +
draw(O--T);
 +
draw(A--P);
 +
draw(Circle(O,9));
 +
 +
dot(A);dot(B);dot(C);dot(T);dot(P);dot(O);dot(Q);
 +
label("\(B\)",B,NW);
 +
label("\(A\)",A,NE);
 +
label("\(\omega\)",O,N);
 +
label("\(P\)",P,S);
 +
label("\(T\)",T,S);
 +
label("\(Q\)",Q,S);
 +
label("\(C\)",C,E);
 +
label("\(\theta\)",C + (-1.5,0), NW);
 +
label("\(9\)", (B+O)/2, N);
 +
label("\(9\)", (O+A)/2, N);
 +
label("\(9\)", (O+T)/2,W);
 +
</asy>
 +
 +
<math>BQ = \omega T + B\omega\sin\theta = 9 + 9\sin\theta = 9(1 + \sin\theta)</math>
 +
 +
<math>QP = BA\cos\theta = 18\cos\theta</math>
 +
 +
<math>BP^2 = BQ^2 + QP^2 = 9^2(1 + \sin\theta)^2 + 18^2\cos^2\theta</math>
 +
 +
<math>BP^2 = 9^2[1 + 2\sin\theta + \sin^2\theta + 4(1 - \sin^2\theta)]</math>
 +
 +
<math>BP^2 = 9^2[5 + 2\sin\theta - 3\sin^2\theta]</math>
 +
 +
Maximum at <math>\sin\theta = \frac { - 2}{ - 6} = \frac {1}{3}</math>
 +
 +
<math>m^2 = 9^2[5 + \frac {2}{3} - \frac {1}{3}] = \boxed{432}</math>
  
 
== See also ==
 
== See also ==

Revision as of 17:16, 23 March 2008

Problem

Let $\overline{AB}$ be a diameter of circle $\omega$. Extend $\overline{AB}$ through $A$ to $C$. Point $T$ lies on $\omega$ so that line $CT$ is tangent to $\omega$. Point $P$ is the foot of the perpendicular from $A$ to line $CT$. Suppose $AB = 18$, and let $m$ denote the maximum possible length of segment $BP$. Find $m^{2}$.

Solution

[asy] size(250); defaultpen(0.70 + fontsize(10)); import olympiad; pair O = (0,0), B = O - (9,0), A= O + (9,0), C=A+(18,0), T = 9 * expi(-1.2309594), P = foot(A,C,T); draw(Circle(O,9)); draw(B--C--T--O); draw(A--P); dot(A); dot(B); dot(C); dot(O); dot(T); dot(P); draw(rightanglemark(O,T,C,30)); draw(rightanglemark(A,P,C,30)); draw(anglemark(B,A,P,35)); draw(B--P, blue); label("\(A\)",A,NW); label("\(B\)",B,NW); label("\(C\)",C,NW); label("\(O\)",O,NW); label("\(P\)",P,SE); label("\(T\)",T,SE); label("\(9\)",(O+A)/2,N); label("\(9\)",(O+B)/2,N); label("\(x-9\)",(C+A)/2,N); [/asy]

Let $x = OC$. Since $OT, AP \perp TC$, it follows easily that $\triangle APC \sim \triangle OTC$. Thus $\frac{AP}{OT} = \frac{CA}{CO} \Longrightarrow AP = \frac{9(x-9)}{x}$. By the Law of Cosines on $\triangle BAP$, \begin{align*}BP^2 = AB^2 + AP^2 - 2 \cdot AB \cdot AP \cdot \cos \angle BAP \end{align*} where $\cos \angle BAP = \cos (180 - \angle TOA) = - \frac{OT}{OC} = - \frac{9}{x}$, so: \begin{align*}BP^2 &= 18^2 + \frac{9^2(x-9)^2}{x^2} + 2(18) \cdot \frac{9(x-9)}{x} \cdot \frac 9x = 405 + 729\left(\frac{2x - 27}{x^2}\right)\end{align*} Let $m = \frac{2x-27}{x^2} \Longrightarrow mx^2 - 2x + 27 = 0$; this is a quadratic, and its discriminant must be nonnegative: $(-2)^2 - 4(m)(27) \ge 0 \Longleftrightarrow m \le \frac{1}{27}$. Thus, \[BP^2 \le 405 + 729 \cdot \frac{1}{27} = \boxed{432}\] Equality holds when $x = 27$.

Alternate Solution

[asy] unitsize(4mm); pair B=(0,13.5), C=(23.383,0); pair O=(7.794, 9), P=(2*7.794,0); pair T=(7.794,0), Q=(0,0); pair A=(2*7.794,4.5);  draw(Q--B--C--Q); draw(O--T); draw(A--P); draw(Circle(O,9));  dot(A);dot(B);dot(C);dot(T);dot(P);dot(O);dot(Q); label("\(B\)",B,NW); label("\(A\)",A,NE); label("\(\omega\)",O,N); label("\(P\)",P,S); label("\(T\)",T,S); label("\(Q\)",Q,S); label("\(C\)",C,E); label("\(\theta\)",C + (-1.5,0), NW); label("\(9\)", (B+O)/2, N); label("\(9\)", (O+A)/2, N); label("\(9\)", (O+T)/2,W); [/asy]

$BQ = \omega T + B\omega\sin\theta = 9 + 9\sin\theta = 9(1 + \sin\theta)$

$QP = BA\cos\theta = 18\cos\theta$

$BP^2 = BQ^2 + QP^2 = 9^2(1 + \sin\theta)^2 + 18^2\cos^2\theta$

$BP^2 = 9^2[1 + 2\sin\theta + \sin^2\theta + 4(1 - \sin^2\theta)]$

$BP^2 = 9^2[5 + 2\sin\theta - 3\sin^2\theta]$

Maximum at $\sin\theta = \frac { - 2}{ - 6} = \frac {1}{3}$

$m^2 = 9^2[5 + \frac {2}{3} - \frac {1}{3}] = \boxed{432}$

See also

2008 AIME I (ProblemsAnswer KeyResources)
Preceded by
Problem 13
Followed by
Problem 15
1 2 3 4 5 6 7 8 9 10 11 12 13 14 15
All AIME Problems and Solutions